Abstract

for any e > 0. This estimate has had many applications, for example to sharpenings of the Brun-Titchmarsh theorem on primes in arithmetic progressions. Burgess' bound (2) is weaker than the trivial bound (1) for q < T 12. However there is an alternative method which improves upon (1) for sufficiently large one treats the q-dependence trivially and applies van der Corput's method to sums of the type

Full Text
Paper version not known

Talk to us

Join us for a 30 min session where you can share your feedback and ask us any queries you have

Schedule a call

Disclaimer: All third-party content on this website/platform is and will remain the property of their respective owners and is provided on "as is" basis without any warranties, express or implied. Use of third-party content does not indicate any affiliation, sponsorship with or endorsement by them. Any references to third-party content is to identify the corresponding services and shall be considered fair use under The CopyrightLaw.